You are on page 1of 15

Page 1 Algebra

ActuaI CAT ProbIens 1998-2008


AIgebra {Answers and ExpIanations}
CAT 1999
1. a The difference between two integers will be 1, only if
one is even and the other one is odd. 4x will always
be even, so 17y has to be odd and hence y has to be
odd.
Moreover, the number 17y should be such a number
that is 1 less than a multiple of 4. In other words, we
have to find all such multiples of 17, which are 1 less
than a multiple of 4. The first such multiple is 51. Now
you will find that as the multiples of 17 goes on
increasing, the difference between it and its closest
higher multiple of 4 is in the following pattern, 0, 3, 2, 1,
e.g. 52 51 = 1,
68 68 = 0, 88 85 = 3, 104 102 = 2, 120 119 = 1,
136 136 = 0
So the multiples of 17 that we are interested in are 3,
7, 11, 15 .
Now since, x 1000, 4x 4000 . The multiple of 17
closest and less than 4000 is 3995 (17 235). And
incidentally, 3996 is a multiple of 4, i.e. the difference
is 4.
This means that in order to find the answer, we need
to find the number of terms in the AP formed by
3, 7, 11, 15 235, where a = 3, d = 4.
Since, we know that Tn = a + (n 1)d,
so 235 = 3 + (n 1) 4. Hence, n = 59.
Alternate Solution:
4x 17y = 1 and x 1000
so, 17y 1 4000 i.e. y 235 + and moreover every 4th
value of y with give value of x.
So, number of values =
235
58
4

So, total number of terms will be 58 + 1 = 59
2. a Let x be the fixed cost and y the variable cost
17500 = x + 25y (i)
30000 = x + 50y (ii)
Solving the equation (i) and (ii), we get
x = 5000, y = 500
Now if the average expense of 100 boarders be A.
Then
100 A = 5000 + 500 100
A = 550.
3. d r 6 11 r 6 11, r 17
or (r 6) = 11, r = 5
|2q 12|=8 2q 12 = 8 , q = 10
or 2q 12 = 8 , q =2
Hence, minimum value of
r
q
=
5
10

= 2.
For questions 4 to 6:
f o e c a l P
p i h s r o w
f o r e b m u N
s r e w o l f
e r o f e b
g n i r e f f o
f o r e b m u N
s r e w o l f
d e r e f f o
f o r e b m u N
s r e w o l f
t f e l
1 y ) 8 / 5 1 ( y y ) 8 / 7 (
2 y ) 4 / 7 ( y y ) 4 / 3 (
3 y ) 2 / 3 ( y 2 / y
4 y y 0
Starting from the fourth place of worship and moving
backwards, we find that number of flowers before
entering the first place of worship is
15
y
8
.
Hence, number of flowers before doubling =
15
y
16
(but this is equal to 30)
Hence, y = 32
Answer for 4 is (c)
The minimum value of y so that
15
y
16
is a whole number
is 16.
Therefore, 16 is the minimum number of flowers that
can be offered.
Answer for 5 is (c).
For y = 16, the value of
15
y 15
16
.
Hence, the minimum number of flowers with which
Roopa leaves home is 15.
Answer for6 is (b).
Page 2
Algebra
7. d Let m = 1. So, option (a) will give the answer as V
m
and option (c) will give the answer as V
1
. Both of
these cannot be the answers as V
m
and V
1
are the
amount of volume filled.
Let m = 2. So, option (b) will give the answer as 2
(1 V
2
) and option (d) will give the answer as
2(1 V
1
). Now consider option (b).
Actual empty volume > 2(1 V
2
). Therefore, for this
situation m(1 V
1
) is the only possible answer.
8. b Let m = 1 and n = 1. Option (a) gives the answer as
4
1
and option (d) gives the answer as greatest integer
less than or equal to
2
1
. So, both of these cannot be
the answer. Option (b) gives the answer as smallest
integer greater than or equal to
2
1
and option (c)
gives the answer as 1. But the actual answer can be
greater than 1 as the volume of the vessel is 2 l.
Hence, (b) is the answer.
CAT 2000
9. b The data is not linear. So check (b).
Let the equation be y = a + bx + cx
2
.
Putting the values of x and y, we get the following
result.
4 = a + b + c, 8 = a + 2b + 4c and 14 = a + 3b + 9c.
Solving these, we get a = 2, b = 1 and c = 1.
So the equation is y = 2 + x + x
2
.
10. d a
1
= 1, a
2
= 7, a
3
= 19, a
4
= 43.
The difference between successive terms is in series
6, 12, 24, 48, ..., i.e. they are in GP. Hence,
( )
( )
99
n
99
100 1
2 1
r 1
a a a 1 6 6 2 5
r 1 2 1
_
+ +

,
11. c
1 1 1 1
...
1.3 3.5 5.7 19.21
+ + + +
1 1 1 1 1 1 1 1 1 1 1
1 ...
2 3 2 3 5 2 5 7 2 19 21
_ _ _ _
+ + +

, , , ,
( ) 21 1 1 1 20 10

2 42 42 42 21

12. c The vehicle travels 19.5 km/L at the rate of 50 km/hr.
So it should travel
19.5
1.3
km/L at the rate of 70 km/hr
= 15 km/L.
The distance covered at 70 km/hr with 10 L = 10 15
13. b Use choices. The answer is (b), because x < 2 and
2 < 2y x < 2y.
14. b Use the choices. If b = 1, then the factors are (x a)
(x
2
+ 1). This cannot yield 3 real roots.
CAT 2001
15. d x > 5, y < 1
Use answer choices.
Take x = 6, y = 6. We see none of the statements
(1, 2 and 3) is true. Hence the correct option is (d).
16. d Let y = n
3
7n
2
+ 11n 5
At n = 1, y = 0
(n 1) (n
2
6n + 5)
= (n 1)
2
(n 5)
Now (n 1)
2
is always positive.
Now for n < 5 the expression gives a negative quantity.
Therefore, the least value of n will be 6.
Hence m = 6.
17. c Let x be the number of males in Mota Hazri.
Chota Hazri Mota Hazri
Males x 4522 x
Females 2(x 4522) x + 4020
x + 4020 2(x 4522) = 2910 x = 10154
Number of males in Chota Hazri = 10154 4522
= 5632
18. b Let the number of students in classes X, Y and Z be a,
b and c respectively. Then
Total of X = 83a
Total of Y = 76b
Total of Z = 85c
And 79
b a
b 76 a 83

+
+
, i.e. 4a = 3b
Also 81
c b
c 85 b 76

+
+
, i.e. 4c = 5b
Hence, b = a
3
4
, c =
5 5 4 5
b a a
4 4 3 3

Average of X, Y and Z =
c b a
c 85 b 76 a 83
+ +
+ +
=
4 5
83a 76 a 85 a
3 3
4 5
a a a
3 3
+ +
+ +
=
5 . 81
12
978

19. a Let the cost of 1 burger, 1 shake and 1 fries be x, y


and z.
Then
3x + 7y + z = 120 ... (i)
4x + 10y + z = 164.5 ... (ii)
x + 3y = 44.5 ... (iii) (ii i)
Multiplying (iii) by 4 and subtracting (ii) from it, we find
2y z = 13.5 ...(iv)
Subtracting (iv) from (iii), we get x + y + z = 31.
Page 3 Algebra
20. c Let the 6th and the 7th terms be x and y.
Then 8th term = x + y
Also y
2
x
2
= 517
(y + x)(y x) = 517 47 11
So y + x = 47
y x = 11
Taking y = 29 and x = 18, we have 8th term = 47,
9th term = 47 + 29 = 76 and 10th term = 76 + 47 = 123.
21. c x + y = 1 and x > 0 y > 0
Taking x = y =
2
1
, value of
2 2
1 1
x y
x y
_ _
+ + +

, ,
2 2
1 1
2 2
2 2
_ _
+ + +

, ,

25 25 25
4 4 2
+
It can be easily verified as it is the least value among
options.
For questions 22 and 23:
1 2 1 2
2
1 1 2
r r r r
BA , MBA
n n n
+ +

+
and
1 2 2 1
1
1 1 2 1
r n r r
MBA max 0,
n n n n

+
' ;

From BA and MBA
2
, we get BA

MBA
2
because
1 2 1
n n n + .
From BA and MBA
1
, we get
1
BA MBA because

'

+ +
1
1
2
2
2
2
1
2
1
1
1
2
1
1
n
r
n
r
, 0 max
r
n
n
r
n
r
n
r
n
r
.
Now from MBA
1
and MBA
2
, we get
2 1
2
2 1
1
1
1
2
2
2
2
1
2
1
1
n n
r
n n
r
n
r
n
r
, 0 max
r
n
n
r
n
r
+
+
+

'

+
.
22. d From the above information,
1 2
BA MBA MBA
None of these is the right answer.
23. b BA = 50 where there is no incomplete innings means r
2
= n
2
= 0 50
n
r
1
1

1
1
]
1

,
_

+
1
1
2
2
1
2
1
1
1
n
r

n
r
, 0 max
n
n
n
r
MBA
=
1
]
1

,
_

+ 50
1
45
, 0 max
n
1
50
1
= 50 + 0 = 50
1 2 1
1 1 1
r r 50n 45 45
BA 50 50
n n n
+ +
+ >
1 2 1
2
1 2 1
r r 50n 45 5
MBA 50
n n n 1 n 1
+ +

+ + +
Hence, BA will increase, MBA
2
will decrease.
24. a Equation of quadratic equation is
ax
2
+ bx + c = 0
x
2
+ bx + c = 0
First roots = (4, 3)
Sum of the roots
b
7 b 7
a

.
Product of the roots
c
12 c 12
a
.
Equation formed x
2
7b + 12 = 0 ... (i)
Another boy gets the wrong roots (2, 3).
Sum of the roots
b
5 b 5
a

.
Product of the roots
c
6 c 6
a
.
Equation formed x
2
5b + 6 = 0 ... (ii)
x
2
+ x b + c
1
= 0
b = 2 + 3
c = 6
Hence, x
2
7x + 6 = 0
x
2
6x x + 6 = 0
x(x 6) 1(x 6) = 0
(x 6)(x 1) = 0
x = 6, 1
Hence, the actual roots = (6, 1).
Alternate method:
Since constant = 6[3 2] and coefficient of
x = [4x 3x] = 7
Since quadratic equation is
x
2
(Sum of roots)x + Product of roots = 0 or
x
2
7x + 6 = 0
Solving the equation (x 6)(x 1) = 0 or x = (6, 1).
CAT 2002
25. 2 f(x) + f(y) = log
1 x 1 y
log
1 x 1 y
_ + + _
+

, ,

(1 x) (1 y)
log
(1 x)(1 y)
_ + +


,

1 x y xy
log
1 xy x y
_ + + +


+
,
Page 4
Algebra

1 xy x y
log
1 xy (x y)
_ + + +


+ +
,

x y
1
1 xy
log
x y
1
1 xy
_ _ +
+

+
,


_ +



+
, ,

x y
f
1 xy
_ +


+
,
26. 4 x
0
= x
x
1
= x
x
2
= x
x
3
= x
x
4
= x
x
5
= x
x
6
= x
..
Choices (1), (2), (3) are incorrect.
27. 3 xy + yz + zx = 3
+ + xy (y x)z 3
+ + xy (y x)(5 x y) 3
+ + +
2 2
x y xy 5x 5y 3 0
+ + +
2 2
y (x 5) y x 5x 3 0
As it is given that y is a real number, the discriminant
for above equation must be greater than or equal to
zero.
Hence,
+
2 2
(x 5) 4(x 5x 3) 0

2
3x 10x 13 0
+
2
3x 13x 3x 13 0
1

1
]
13
x 1,
3
Largest value that x can have is
13
3
.
28. 1 Coefficient of
n
1
x (n 1)(n 4)
2
+ +
2 3
2
2 3
S 2 5x 9x 14x ....
xS 2x 5x .....
S(1 x) 2 3x 4x 5x ....
+ + + +
+ +
+ + + +
Let
1
S S(1 x)
2
1
S 2 3x 4x ... + + +
2
1
2
1
xS 2x 3x ...
S (1 x) 2 x x ....
+ +
+ + +
1
x
S (1 x) 2
1 x
+

2
x
S(1 x ) 2
1 x
+


3
2 x
S
(1 x)

29. 3
2 2 2
x 5y z 4yx 2yz + + +
2 2 2 2
(x 4y 4yx) z y 2yz 0 + + +
2 2
(x 2y) (z y) 0 +
It can be true only if x = 2y and z = y
30. 2 Arithmetic mean is more by 1.8 means sum is more by
18. So ba ab = 18
b > a because sum has gone up, e.g. 31 13 = 18
Hence, b a = 2
31. 4 x 1 [x] x
2x 2y 3 L(x, y) 2x 2y + + a 3 L a
2x 2y 2 R(x, y) 2x 2y + + a 2 R a
Therefore, L R
Note: Choice (2) is wrong, otherwise choice (1) and
choice (3) are also not correct. Choose the numbers
to check.
32. 4
2 2
A B
1
x x 1
+
2 2 2
A (x 1) B x x x +
This is a quadratic equation.
Hence, number of roots = 2 or 1 (1 in the case when
both roots are equal).
33. 3 Let the largest piece = 3x
Middle = x
Shortest = 3x 23
or 3x + x + (3x 23) = 40
or x = 9
or the shortest piece = 3(9) 23 = 4
Check choices:
The shortest piece has to be < 20 cm.
27 is wrong choice.
The largest piece is a multiple of 3.
Or (23 + Shortest) should be a multiple of 3.
Answer = 4 cm (Among other choices)
34. 3 If p = q = r = 1, then expression = 1
Check the choice only, one choice gives the value of
expression = 1.
CAT 2003 (Leak)
35. 3 2
x
x 1 = 0
2
x
1 = x
If we put x = 0, then this is satisfied and if we put
x = 1, then also this is satisfied.
Now we put x = 2, then this is not valid.
Page 5 Algebra
36. 2 For the curves to intersect, log
10
x = x
1
Thus,
x
10
1
log x or x 10
x

This is possible for only one value of x (2 < x < 3).
37. 1 It is given that p q r 0 + + , if we consider the first
option, and multiply the first equation by 5, second by
2 and third by 1, we see that the coefficients of x,
y and z all add up-to zero.
Thus, 5p 2q r = 0
No other option satisfies this.
38. 1 Let 'x' be the number of standard bags and 'y' be the
number of deluxe bags.
Thus, 4x + 5y 700 and 6x + 10y 1250
Among the choices, (3) and (4) do not satisfy the
second equation.
Choice (2) is eliminated as, in order to maximize profits
the number of deluxe bags should be higher than the
number of standard bags because the profit margin is
higher in a deluxe bag.
39. 3 Let the 1st term be a and common difference be d
then we have 3
rd
term = a + 2d
15
th
term = a + 14d
6
th
term = a + 5d
11
th
term = a + 10d
13
th
term = a + 12d
Since sum of 3
rd
and 15
th
term = sum of 6
th
, 11
th
and
13
th
term, therefore we have
2a + 16d = 3a + 27d
a + 11d = 0
Which is the 12
th
term.
40. 4 We can see that x + 2 is an increasing function and
5 x is a decreasing function. This system of equation
will have smallest value at the point of intersection of
the two. i.e. 5 x = x + 2 or x = 1.5.
Thus smallest value of g(x) = 3.5
41. 2 Case 1: If x < 2, then y = 2 x + 2.5 x + 3.6 x
= 8.1 3x.
This will be least if x is highest i.e. just less than 2.
In this case y will be just more than 2.1
Case 2: If 2 x 2.5 < , then y = x 2 + 2.5 x +
3.6 x = 4.1 x
Again, this will be least if x is the highest i.e. just less
than 2.5. In this case y will be just more than 1.6.
Case 3: If 2.5 x 3.6 < , then y = x 2 + x 2.5 + 3.6
x = x 0.9
This will be least if x is least i.e. x = 2.5.
Case 4: If x 3.6 , then
y = x 2 + x 2.5 + x 3.6 = 3x 8.1
The minimum value of this will be at x = 3.6 and y = 2.7
Hence the minimum value of y is attained at x = 2.5
Alternate method:
At x = 2, f(x) = 2.1
At x = 2.5, f(x) = 1.6
At x = 3.6, f(x) = 2.7
Hence, at x = 2.5, f(x) will be minimum.
42. 2 Solution can be found using Statement A as we know
both the roots for the equation (viz.
1
2
and
1
2

).
Also statement B is sufficient.
Since ratio of c and b = 1, c = b.
Thus the equation = 4x
2
+ bx + b = 0. Since x =
1
2

is
one of the roots, substituting we get 1
b
2
+ b = 0 or
b = 2. Thus c = 2.
43. 1 Both the series are infinitely diminishing series.
For the first series: First term =
2
1
a
and r =
2
1
a
For the second series: First term =
1
a
and r =
2
1
a
The sum of the first series =
2
2
2
1
1
a
1
a 1
1
a

The sum of the second series =


2
2
1
a
a
1
a 1
1
a

Now, from the first statement, the relation can be


anything (depending on whether a is positive or
negative).
But the second statement tells us, 4a
2
4a + 1 = 0 or
a =
1
2
. For this value of a, the sum of second series
will always be greater than that of the first.
44. 4 The number of terms of the series forms the sum of
first n natural numbers i.e.
n(n 1)
2
+
.
Thus the first 23 letters will account for the first
23 24
2

= 276 terms of the series.


The 288
th
term will be the 24
th
letter which is x.
Page 6
Algebra
45. 4 p + q = 2 and pq = 1
(p + q)
2
= p
2
+ q
2
+ 2pq,
Thus ( 2)
2
= p
2
+ q
2
+ 2( 1)
p
2
+ q
2
=
2
4 + 4 + 2 + 2
p
2
+ q
2
=
2
2 + 6
p
2
+ q
2
=
2
2 + 1 + 5
p
2
+ q
2
= ( 1)
2
+ 5
Thus, minimum value of p
2
+ q
2
is 5.
46. 2 (a + b + c + d)
2
= (4m + 1)
2
Thus, a
2
+ b
2
+ c
2
+ d
2
+ 2(ab + ac + ad + bc + bd + cd)
= 16m
2
+ 8m + 1
a
2
+ b
2
+ c
2
+ d
2
will have the minimum value if (ab + ac
+ ad + bc + bd + cd) is the maximum.
This is possible if a = b = c = d = (m + 0.25) since a
+ b + c + d = 4m + 1
In that case 2((ab + ac + ad + bc + bd + cd)
= 12(m + 0.25)
2
= 12m
2
+ 6m + 0.75
Thus, the minimum value of a
2
+ b
2
+ c
2
+ d
2
= (16m
2
+ 8m + 1) 2(ab + ac + ad + bc + bd + cd)
= (16m
2
+ 8m + 1) (12m
2
+ 6m + 0.75)
= 4m
2
+ 2m + 0.25
Since it is an integer, the actual minimum value
= 4m
2
+ 2m + 1
47. 3 Assume the number of horizontal layers in the pile be
n.
So
n(n 1)
8436
2
+

2
1
[ n n] 8436
2
+
n(n 1) (2n 1) n(n 1)
8436
12 4
+ + +
+
2n 4
n(n 1) 8436
12
+ 1
+
1
]
n(n 1)(n 2)
8436
6
+ +

n(n 1) (n 2) 36 37 38 + +
So n = 36
48. 3 Using log a log b =
a
log
b
_

,
,
2 y 5
y 5 y 3.5


, where
y = 2
x
Solving we get y = 4 or 8 i.e. x = 2 or 3. It cannot be 2
as log of negative number is not defined (see the
second expression).
49. 2 u is always negative. Hence, for us to have a minimum
value of
vz
u
, vz should be positive. Also for the least
value, the numerator has to be the maximum positive
value and the denominator has to be the smallest
negative value. In other words, vz has to be 2 and u
has to be 0.5.
Hence the minimum value of
vz
u
=
2
0.5
= 4.
For us to get the maximum value, vz has to be the
smallest negative value and u has to be the highest
negative value. Thus, vz has to be 2 and u has to be
0.5.
Hence the maximum value of
vz
u
=
2
0.5

= 4.
50. 2 GRRRRR, RGRRRR, RRGRRR, RRRGRR, RRRRGR,
RRRRRG
GGRRRR, RGGRRR, RRGGRR, RRRGGR, RRRRGG
GGGRRR, RGGGRR, RRGGGR, RRRGGG
GGGGRR, RGGGGR, RRGGGG
GGGGGR, RGGGGG
GGGGGG
Hence 21 ways.
51. 4 When we substitute two values of x in the above
curves, at x = 2 we get
y = 8 + 4 + 5 = 1
y = 4 2 + 5 = 7
Hence at x = 2 the curves do not intersect.
At x = 2, y
1
= 17 and y
2
= 11
At x = 1, y
1
= 5 and 2 and y
2
= 5
When x = 0, y
1
= 5 and y
2
= 5
And at x = 1, y
1
= 7 and y
2
= 7
Therefore, the two curves meet thrice when x = 1, 0
and 1.
52. 1 Let us say there are only 3 questions. Thus there are
2
31
= 4 students who have done 1 or more questions
wrongly, 2
32
= 2 students who have done 2 or more
questions wrongly and 2
33
= 1 student who must
have done all 3 wrongly. Thus total number of wrong
answers = 4 + 2 + 1 = 7 = 2
3
1 = 2
n
1.
In our question, the total number of wrong answers
= 4095 = 2
12
1. Thus n = 12.
53. 3 Here x, y, z are distinct positive real number
So
2 2 2
x (y z) y (x 2) z (x y)
xyz
+ + + + +
x x y y z z
y z x z x y
+ + + + +
x y y z z x
y x z y x z
_ _ _
+ + + + +

, , ,
[We know that
a b
2
b a
+ > if a and b are distinct numbers
> 2 + 2 + 2
> 6
Page 7 Algebra
54. 1 The least number of edges will be when one point is
connected to each of the other 11 points, giving a total
of 11 lines. One can move from any point to any other
point via the common point. The maximum edges will
be when a line exists between any two points. Two
points can be selected from 12 points in
12
C
2
i.e. 66
lines.
CAT 2003 (Re-TEST)
55. 2 Consider first zone. The number of telephone lines
can be shown a follows.
= 9 lines
Therefore, total number of lines required for internal
connections in each zone = 9 4 = 36 lines.
Now consider the connection between any two
zones.
Each town in first zone can be connected to three
towns in the second zone.
Therefore, the lines required = 3 3 = 9
Therefore, total number of lines required for connecting
towns of different zones =
4
C
2
9 = 6 9 = 54
Therefore, total number of lines in all = 54 + 36 = 90
56. 2 ax
2
+ bx + 1 = 0
For real roots
2
b 4ac 0
2
b 4a(1) 0
2
b 4a
For a = 1, 4a = 4, b = 2, 3, 4
a = 2, 4a = 8, b = 3, 4
a = 3, 4a = 12, b = 4
a = 4, 4a = 16, b = 4
Number of equations possible = 7.
57. 2
10 10 x
log x log x 2log 10
10 x
x
log log 100
x
1

1
]
10
10
10
log 100
log x
log x

10
10
1 2
log x
2 log x

( )
2
10
log x 4
10
log x 2 t
10 10
log x 2 or log x 2
2 2
10 x or 10 x


1
x 100 or x
100

58. 2
3 3 0.008
1
log M 3log N 1 log 5
3
+ +
( )
1/ 3 3
3
log10 log2
log (M N ) 1
log8 log1000
+
( )
( )
1/ 3 3
3
1 log2
log (M N ) 1
3 1 log2

1/ 3 3
3
1 2
log (M N ) 1
3 3

M
1/3
N
3
= 3
2/3
MN
9
= 3
2
N
9
= 9/M.
59. 3 5x + 19y = 64
We see that if y =1, we get an integer solution for
x = 9, now if y changes (increases or decreases) by
5x will change (decrease or increase) by 19.
Looking at the options, if x = 256, we get y = 64.
Using these values we see options (1), (2) and (4)
are eliminated and also that these exists a solution for
250 < x 300.
60. 4 Sum of
2 3
2
m m
log m log log
n
n
_ _

+ + +

, ,
n terms such
problem must be solved by taking the value of number
of terms. Lets say 2 and check the given option. If we
look at the sum of 2 terms of the given series it comes
out to be
2 2 3
m m m m
log m log log log
n n n
_


+

,
Now look at the option and put number of terms as 2,
only option (4) validates the above mentioned answer.
As
n
1
(n 1) 3 3
2
(n 1)
m m m
log log log
n n
n
+

1 1 _
1 1

1 1
] ] ,
Page 8
Algebra
61. 2 xyz = 4
y x = z y
2y = x + z
y is the AM of x, y, z.
Also
2
3 3
xyz 4
1
3 3
xyz 2
AM GM
2
3
y 2
Therefore, the minimum value of y is
2
3
2 .
62. 3 Let
2 3 4
4 9 16 25
S 1
7
7 7 7
+ + + + (i)

2 3 4
1 1 4 9 16
S
7 7
7 7 7
+ + +
(ii)
(i) (ii) gives,

2 3 4
1 3 5 7 9
S 1 1
7 7
7 7 7
_
+ + + +

,
(iii)

2 3 4
1 1 1 3 5 7
S 1
7 7 7
7 7 7
_
+ + +

,
(iv)
(iii) (iv) gives,
2 3 4
1 1 1 2 2 2 2
S 1 S 1 1
7 7 7 7
7 7 7
_ _
+ + + +

, ,

2
1 1 2 1 1
S 1 1 1 1
7 7 7 7
7
_ _ 1
+ + + +
1
, , ]

2
1 2 1
S 1 1
1
7 7
1
7
_
+

,

2
6 2 7
S 1
7 7 6
_
+

,
36 1
S 1
49 3
+
49 4
S
36 3

49
S
27

63. 1 Let is the common root.


3 2
3 4 5 0 + + +
3 3
2 7 3 0 + + +
2
3 2 0 +
2, 1
But the above values of do not satisfy any of the
equations. Thus, no root is common.
64. 3
1 1
1 x 3
n n
< +
Put n = 1
0 x 4 <
65. 4 36 72 n
2
n 2 n(n 4) 16
x
n 4 n 4
+ + +

+ +
Put x = 36.
2
(36) 2 6 40 16
x
36 24 4
+ +

+ +
Which is least value of n = 28
66. 4 13x + 1 < 2z and z + 3 =
2
5y
13x + 1 < 2
2
(5y 3)
2
13x 1 10y 6 + <
2
13x 7 10y + <
put x = 1
2
20 10y <
2
y 2 >
2
y 2 >
2
(y 2) 0 >
X = 1
2
0
y
2
67. 2 x = |a| b
Now a xb = a ( |a| b) b
= a + |a| b
2
a xb = a + ab
2
a 0 OR a xb
= a ab
2
a < 0
= a(1 + b
2
) = a(1 b
2
)
Consider first case:
As a 0 and |b| 1, therefore (1 + b
2
) is positive.
a (1 + b
2
) 0
a xb 0
Consider second case.
As a < 0 and |b| 1, therefore (1 b
2
) 0
a (1 b
2
) 0 (Since ve -ve = +ve and 1 b
2
can
be zero also), i.e. a xb 0
Therefore, in both cases a xb 0.
Page 9 Algebra
68. 1
2
g g g h
3 2
g g g h g f
4 3
g g g f g e
n 4
69. 4 f [f {f (f f )}]
= f [f {f h}]]
= f [f e}]
= f [f ]]
= h
70. 1
8 2 2 2
e e e e
= e e e
= e
If we observe a

anything = a
a
10
= a
10 10 9 8
{a (f g )} e
= a e
= e
71. 4 It will go by elimination.
9 7 = 2 is even, therefore option (1) not possible.
2 9 = 18 is even, therefore option (2) not possible.
3 9 12
4
3 3
+

is even, therefore option (3) is not
possible.
The correct option is (4).
CAT 2004
72. 1 Given
1 2 11 1 2 19
t t ... t t t .... t + + + + + + ( for an A.P.)
[ ] [ ]
11 19
2a (11 1)d 2a (19 1)d
2 2
+ +
22 a + 110d = 28 a + 342 d
16 a + 232 d = 0
2a + 29 d = 0
[ ]
30
2a (30 1)d 0
2
+
30terms
S 0
73. 2 We have
3
f(0) 0 4(0) p p +
3
f(1) 1 4(1) p p 3 +
If P and P 3 are of opp. signs then p(p 3) < 0
Hence 0 < p < 3.
74. 1 We have
(1) 10
10
< n < 10
11
(2) Sum of the digits for 'n' = 2
Clearly-
(n)min = 10000000001 (1 followed by 9 zeros and
finally 1)
Obviously, we can form 10 such numbers by shifting
'1' by one place from right to left again and again.
Again, there is another possibility for 'n'
n = 20000000000
So finally : No. of different values for n = 10 + 1 = 11
ans.
75. 3 If
a b c
r
b c c a a b

+ + +
then there are only two possibilities.
(i)
If a b c 0, then + +
a b c a b c
b c c a a b (b c) (c a) (a b)
+ +

+ + + + + + + +
a b c 1
2(a b c) 2
+ +

+ +
(ii)
If a+ b + c = 0, then
b + c = a
c + a = b
a + b = c
Hence
a a
1
b c (a)

+
Similarly,
b c
1
c a a b

+ +
Therefore option (3) is the correct one 1/2 or 1
76. 4
1
y
1
2
3 y

+
+
3 y
y
7 2y
+

+

2
2y 6y 3 0 +

6 36 24
y
4
t +


6 60 3 15
4 2
t t

Since 'y' is a +ve number, therefore:
15 3
y
2

ans.
Page 10
Algebra
77. 4 When a > 0, b < 0,
ax
2
and b |x| are non negative for all x,
i.e. ax
2
b|x| 0
ax
2
b |x| is minimum at x = 0 when a > 0, b < 0.
78. 4
Family Adults Children
1 0, 1, 2 3, 4, 5, .
II 0, 1, 2 3, 4, 5, .
III 0, 1, 2 3, 4, 5, .
As per the question, we need to satisfy three
conditions namely:
1. Adults (A) > Boys (B)
2. Boys (B) > Girls (G)
3. Girls (G) > Families (F)
Clearly, if the number of families is 2, maximum number
of adults can only be 4. Now, for the second condition
to be satisfied, every family should have atleast two
boys and one girl each. This will result in non-
compliance with the first condition because adults will
be equal to boys. If we consider the same conditions
for 3 families, then all three conditions will be satisfied.
79. 3 Given equation is x + y = xy
xy x y + 1 = 1
(x 1)(y 1) = 1
x 1 1& y 1 1or x 1 1& y 1 1
Clearly (0, 0) and (2, 2) are the only pairs that will
satisfy the equation.
80. 3 Given a
1
= 81.33; a
2
= 19
Also:
a
j
= a
j1
a
j2
, for j 3
a
3
= a
2
a
1
= 100.33
a
4
= a
3
a
2
= 81.33
a
5
= a
4
a
3
= 19
a
6
= a
5
a
4
= +100.33
a
7
= a
6
a
5
= +81.33
a
8
= a
7
a
6
= 19
Clearly onwards there is a cycle of 6 and the sum of
terms in every such cycle = 0. Therefore, when we
add a
1
, a
2
, a
3
... upto a
6002
, we will eventually be left with
a
1
+ a
2
only i.e. 81.33 19 = 62. 33.
81. 2 u = (log
2
x)
2
6log
2
x + 12
x
u
= 256
Let log
2
x = y x = 2
y
u 8
8
x 2 uy 8 u
y


2 3 2
8
y 6y 12 y 6y 12y 8 0
y
+ +

3
(y 2) 0 y 2
x 4, u 4
82. 1 Since Group (B) contains 23 questions, the marks
associated with this group are 46.
Now check for option (1). If Group (C ) has one
question, then marks associated with this group will
be 3. This means that the cumulative marks for these
two groups taken together will be 49. Since total
number of questions are 100, Group (A) will have 76
questions, the corresponding weightage being 76
marks. This satisfies all conditions and hence is the
correct option. It can be easily observed that no other
option will fit the bill.
83. 3 Since Group (C) contains 8 questions, the
corresponding weightage will be 24 marks. This figure
should be less than or equal to 20% of the total marks.
Check from the options . Option (3) provides 13 or 14
questions in Group (B), with a corresponding
weightage of 26 or 28 marks. This means that number
of questions in Group (A) will either be 79 or 78 and
will satisfy the desired requirement.
CAT 2005
84. 4 R =
( )
( )
65
65 65
65 65
64 64 64
64 64
1
30 30 1
30 30 1
30
30 30 1 1
30 30 1
30
_



,

+ _
+

,
or R =
65
65
64 64
1
1 1
36 30
30 1
1 1
30

_



,
' ;
_
+

,
R = 30
( )
( )
65
64
1 0.96
1 0.96


' ;
+

in
( )
( )
65
64
1 0.96
1 0.96

+
N
r
is only slightly less then 1.
& D
r
is only slightly more than 1. R is slightly less than
36 but certainly greater than 1.
85. 4 If p = 1! = 1
Then p + 2 = 3 when divided by 2! remainder will be 1.
If p = 1! + 2 2! = 5
Then p + 2 = 7 when divided by 3! remainder is still 1.
Hence p = 1! + (2 2!) + (3 3!) + + (10 10!) when
divided by 11! leaves remainder 1
Alternative method:
P = 1 + 2.2! + 3.3!+ .10.10!
= (2 1)1! + ( 3 1)2! + (4 1)3! + .(11 1)10!
=2! 1! + 3! 2! + .. 11! 10! = 1 + 11!
Hence the remainder is 1.
Page 11 Algebra
86. 3 a
1
= 1, a
n+1
3a
n
+ 2 = 4n
a
n+1
= 3a
n
+ 4n 2
when n = 2 then a
2
= 3 + 4 2 = 5
when n = 3 then a
3
= 3 5 + 4 2 2 = 21
from the options, we get an idea that a
n
can be
expressed in a combination of some power of
3 & some multiple of 100.
(1) 3
99
200; tells us that a
n
could be: 3
n1
2 n;
but it does not fit a
1
or a
2
or a
3
(2) 3
99
+ 200; tells us that a
n
could be: 3
n1
+ 2 n;
again, not valid for a
1
, a
2
etc.
(3) 3
100
200; tells 3
n
2n: valid for all a
1
, a
2
, a
3
.
(4) 3
100
+ 200; tells 3
n
+ 2n: again not valid.
so, (3) is the correct answer.
87. 4 P = x y
x y
log log
y x
_ _
+

, ,
=
x x y y
log x log y log y log x +
=
x y
2 log y log x
Let,
x
t l og y
2
1 1
p 2 t t
t t
1

1
]
Which can never be positive, out of given option it
cant assume a value of +1. So (4) is ans.
88. 3
2
x 4 4 x x 4 4 x + +
( )
2
x 4 4 x
Now put the values from options.
Only 3
rd
option satisfies the condition.
89. 4 There are two equations to be formed 40 m + 50 f
= 1000
250 m + 300 f + 40 15 m + 50 10 f = A
850 m + 8000 f = A
m and f are the number of males and females A is
amount paid by the employer.
Then the possible values of f = 8, 9, 10, 11, 12
If f = 8, M = 15
If f = 9, 10, 11 then m will not be an integer while f = 12
then m will be 10.
By putting f = 8 and m = 15, A = 18800. When f = 12 and
m = 10 then A = 18100
Therefore the number of males will be 10.
CAT 2006
90. 1 t
3
t
4
t
5
....t
53

3 4 5 51 52 53 3 4 2
.......
5 6 7 53 54 55 54 55 495
Hence option (1).
91. 1
+
2/ 3 1/ 3
x x 2 0

+
2/ 3 1/ 3 1/ 3
x 2x x 2 0
( )( )
+
1/ 3 1/ 3
x 1 x 2 0


1/ 3
2 x 1
8 x 1
92. 4 Let number of elements in progression be n, then
( ) + 1000 1 n 1 d
( )
3
n 1 d 999 3 37
Possible values of d = 3, 37, 9, 111, 27, 333, 999
Hence 7 progressions.
93. 4 From the graph of (y - x) Vs. (y + x), it is obvious that
inclination is more than 45.
Slope of line =

+
+
y x
tan(45 );
y x
+

+
y x 1 tan
y x 1 tan
By componendo-dividendo,
+

+
y x 1 tan
y x 1 tan
which
is nothing but the slope of the line that shows the
graph of y Vs. x.
And as 0< < 45, absolute value of tan is less
than 1.

1
tan
is negative and also, greater than 1.
The slope of the graph y Vs. x must be negative
and greater than 1. Accordingly, only option (4)
satisfies.
You can also try by putting the values of (y + x) =
2(say) and (y - x) = 4(anything more than 2 for that
matter). You can solve for values of y and x and
cross check with the given options.
Alternate method:
In the normal X-Y coordinate plane the X-axis
corresponds to y = 0
And Y-axis corresponds to x = 0
y + x = 0 and y - x = 0 are perpendicular lines on this
plane.
y
x
y+x=0
(YX) axis
(yx)=0
(Y+X) axis
Page 12
Algebra
And y-x = 0 is the axis Y+X and y+x = 0 is the axis
Y-X
So, the dotted line is the graph drawn in the question.
When you observe w.r.t to X-axis it looks like
x
y
94. 2 2x + y = 40
x y
y = 40 2x
Values of x and y that satisfy the equation
x y
1 38
2 36
3 34
. .
. .
. .
13 14
13 values of (x, y) satisfy the equation such that x
y
95. 5
y x z
y z x
log a.log b.log a b

x x
y y
y y
z z
log log
a and b
log log
_



,
x x
y y
y y
z z
log log
a b
log log
{
_ _


, ,

_ _


, ,
x x
k k
y y
k k
y y
k k
z z
k k
log log
log log
For some base k
log log
log log
( )
( )
_



,
3
x
3
3 x k
y
y
k
log
log ab
log
So,

3 3
ab a b 0
Or, ( )

2 2
a b 1 a b 0
t ab 1
Only option (5) does not satisfy.
Hence (5).
96. 5 Equation (ii) can be written as
( )
1/ 5 0.3x 0.2y
4 9 8 81

2 0.3x 2 0.2y 1/ 5
(2 ) (3 ) 8 (81)

0.6x 0.4y 3 4 1/ 5 3 4 / 5
2 3 2 (3 ) 2 3
0.6x = 3 x = 5
and 0.4y

4
5
y =2
If we put the values of x and y in first equation these
values satisfy the first equation also.
So the answer is x = 5, y = 2
Hence, option (5)
97. 5 f(x) = max (2x + 1, 3 - 4x)
So, the two equations are y = 2x + 1 and y = 3 - 4x
y 2x = 1
+

y x
1
1 1/ 2
Similarly y + 4x = 3
+
y x
1
3 3/ 4
Their point of intersection would be
2x + 1 = 3 - 4x
6x = 2

1
x
3
(1/3,5/3)
(0,3)
(-1/2,0) (0,3/4)
y=34x
y=2x+1
x
y
Page 13 Algebra
So, when

max
1
x then f(x) 3 4x
3
And when
+
max
1
x then f(x) 2x 1
3
Hence the min. of this would be at x =
1
3
i.e.

5
y
3
Alternative method:
As f(x) = max (2x + 1, 3 - 4x)
We know that f(x) would be min at the point of
intersection of these curves
i.e. 2x + 1 = 3 - 4x
6x = 2
i.e.

1
x
3
Hence min f(x) is
5
3
CAT 2007
98. 1 f(1) + f(2) + f(3) + . + f(n) =
2
n f(n)
, f(1) = 3600.
For n = 2:
f(1) + f(2) =
2
2 f(2)
f(2) =
2
f(1)
(2 1)
For n = 3:
( )
2
2
1
3600 1 f(3) 3 f(3)
2 1
_

+ +


,
2
2 2
2 1
f(3) 3600
2 1 3 1
_
_



,
,
Similarly
( )( )( ) ( )
2 2 2 2
2 2 2 2
2 3 4 ... 8
f(9) 3600
2 1 3 1 4 1 ... 9 1


Therefore, f(9) = 80
99. 3 Let the number of currency 1 Miso, 10 Misos and 50
Misos be x, y and z respectively.
x + 10y + 50z = 107
Now the possible values of z could be 0, 1 and 2.
For z = 0: x + 10y = 107
Number of integral pairs of values of x and y that
satisfy the equation x + 10y = 107 will be 11. These
values of x and y in that order are
(7, 10); (17, 9); (27, 8)
(107, 0).
For z = 1: x + 10y = 57
Number of integral pairs of values of x and y that
satisfy the equation x + 10y = 57 will be 6. These
values of x and y in that order are (7, 5); (17, 4); (27,
3);
(37, 2); (47, 1) and (57, 0).
For z = 2: x + 10y = 7
There is only one integer value of x and y that
satisfies the equation x + 10y = 7 in that order is
(7, 0).
Therefore total number of ways in which you can
pay a bill of 107 Misos = 11 + 6 + 1 = 18
100. 5
1 4 1
, n 60
m n 12
+ <
1 1 4 n 48

m 12 n 12n

12n
m
n 48

Positive integral values of m for odd integral values of
n are for n = 49, 51 and 57.
Therefore, there are 3 integral pairs of values of m
and n that satisfy the given equation.
101. 1 Using A: x = 30, y = 30 and z = 29 will give the
minimum value.
Using B: Nothing specific can be said about the relation
between x, y and z.
Hence option (1) is correct choice.
102. 2 Using the given data
( ) ( )
2 2
2
240 40b 40 c 240 20b 20 c
2
3
240 20b 20 c
+ + + +

+ +
and
( ) ( )
2 2
2
240 60b 60 c 240 40b 40 c
1
2
240 40b 40 c
+ + + +

+ +
Solving the above equations
1
c and b 10
10

So, cost for producing x units =
2
x
240 10x
10
+ +
Profit earned from x units
=
2 2
x x
30x 240 10x 20x 240
10 10
_
+ +

,
For maximum profit 20 =
2x
10
So, x = 100.
Page 14
Algebra
103. 4 Profit, in rupees =
2
x
20x 240
10
For x = 100
Profit, in rupees = 760
104. 2 Let f(x) =
2
ax bx c + +
At x = 1, f(1) = a + b + c = 3
At x = 0, f(0) = c = 1
The maximum of the function f(x) is attained at
x =
b

2a
= 1 =
a 2
2a
a = 2 and b = 4
Therefore f(x) =
2
2x + 4x + 1
Therefore f(10) = 159
For questions 105 and 106:
Using the given expressions
1 1
2
2 2
2 2
3 3
2 2 3
4 4
3 2 2 3
5 5
3 3 2 4
6 6
a p b q
a pq b q
a p q b pq
a p q b pq
a p q b p q
a p q b p q






and so on
105. 1
( )
n n n n
1 1
2 2 2 2
n n
a b n is even p q p q
+
+ +
( ) ( )
n1
2
q pq p q +
106. 4
( )
n 1 n 1 n 1 n 1
2 2 2 2
n n
a b n is odd p q p q
+ +
+ +
( )( )
n1
2
p q pq +
Substituting
1 2
p and q
3 3

n 1
2
n n
2
a b
9

_
+

,
Substituting n = 7,
n n
a b 0.01 + >
Substituting n = 9,
n n
a b 0.01 + <
Hence smallest value of n is 9
CAT 2008
107. 2 x
3
ax
2
+ bx c = 0
Let the roots of the above cubic equation be
( 1), , ( + 1)
( 1) + ( + 1) + ( + 1) ( 1) = b

2
+
2
+ +
2
1 = b 3
2
1 = b
Thus, the minimum possible value of b will be equal
to 1 and this value is attained at = 0.
108. 2 Given that f(x) = ax
2
+ bx + c
Also, f(5) = 3f(2) f(5) + 3f(2) = 0
(25a + 5b + c) + 3(4a + 2b + c) = 0
37a + 11b + 4c = 0 (i)
Also, as 3 is a root of f(x) = 0, therefore f(3) = 0.
Therefore, 9a + 3b + c = 0 (ii)
Using equation (i) and (ii), we get that a = b
Therefore, c = 12a
f(x) = a(x
2
+ x 12) = a(x + 4) (x 3)
Therefore, the other root of f(x) = 0 is 4.
109. 5 f(x) = a(x
2
+ x 12)
Therefore, the value of a + b + c cannot be uniquely
determined.
110. 3 Total number of terms in the sequence 17, 21, 25
417 is equal to
417 17
1 101
4

+ .
Total number of terms in the sequence 16, 21, 26
466 is equal to
466 16
1 91
5

+
.
n
th
term of the first sequence = 4n + 13.
m
th
term of the second sequence = 5m + 11.
As per the information given in the question 4n + 13
= 5m + 11
5m 4n = 2.
Possible integral values of n that satisfy 5m = 2 + 4n
are (2, 7, 12 97)
Therefore, the total number of terms common in both
the sequences is 20.
111. 2 f(x).f(y) = f(xy)
Given, f(2) = 4
We can also write;
f(2) = f(2 1) = f(2) f(1)
OR f(1) 4 = 4
f(1) = 1
Now we can also write,
1 1
f(1) f 2 f(2) f
2 2
_ _


, ,
OR
1 f(1) 1
f
2 f(2) 4
_


,
OR
1 1
f
2 4
_


,
Page 15 Algebra
112. 5 seed(n) function will eventually give the digit-sum of
any given number, n.
All the numbers n for which seed(n) = 9 will give the
remainder 0 when divided by 9.
For all positive integers n, n < 500, there are 55 multiples
of 9.
113. 1
2 2 2 2 2 2
1 1 1 1 1 1
S 1 1 ... 1
1 2 2 3 2007 2008
+ + + + + + + + +
n
2 2
1 1
T 1
n (n 1)
+ +
+

4 3 2
2 2
n 2n 3n 2n 1
n (n 1)
+ + + +

+
=
2
2 2
n n 1 1
1
n n n n
+ +
+
+ +

2007 2007
n
n 1 n 1
1 1
S T 2007
n n 1


+
' ;
+


1
2008
2008

You might also like